A family of sets that is not a $sigma$-algebra











up vote
1
down vote

favorite












Let $X$ be a not empty set.



I must prove that the family $mathcal{A}$ defined as follows
$$mathcal{A}=bigg{Ainmathcal{P}(X);|;A;text{is finite or $A^c$ is finite}bigg}$$ it is not a $sigma$-algebra if $X$ is infinite.



Obviously $mathcal{A}$ is an algebra. To show that it is not a $sigma$-algebra I reasoned as follows: fixed a sequence ${x_n}_{ninmathbb{N}}$ of elements of $X$ such that $x_ine x_j$ for $ine j$ and let $A_n$ be the set consisting of the single point $x_n$, in symbols $A_n={x_n}$ for all $ninmathbb{N}$. At this point I consider the sets $A_2,A_4dots ,A_{2k},dots$ where $kinmathbb{N}$; I observe that $A_{2k}inmathcal{A}$ for all $kinmathbb{N}$, but the union $$bigcup_{k=1}^{+infty}A_{2k}=bigg{x_2,x_4,dots,x_{2k},dotsbigg}$$
is infinite. On the other hand
begin{equation}
begin{split}
bigg[bigcup_{k=1}^{+infty}A_{2k}bigg]^c=Xsetminusbigcup_{k=1}^{+infty}A_{2k}=& Xcapbigg[bigcup_{k=1}^{+infty}A_{2k}bigg]^{c}=Xcapbigg[bigcup_{k=1}^{+infty}A_{2k+1}bigg]\
=&bigcup_{k=1}^{+infty}A_{2k+1}cuptilde{A},
end{split}
end{equation}

where $tilde{A}$ it is what remains if $X$ is not countable, if $X$ is countable $tilde{A}=emptyset$. Therefore $cup_k A_{2k}$ and $[cup_k A_{2k}]^c$ are both infinite, then $$bigcup_{n=1}^{+infty} A_nnotinmathcal{A}.$$



it's correct? Thanks!










share|cite|improve this question






















  • Your solution is perfectly fine, you are using that a countable collection of sets in a sigma algebra stays within the sigma algebra, which is part of the definition of a sigma algebra and you are right in proving that this doesn’t hold here
    – Sorin Tirc
    Nov 18 at 13:52










  • Your argument is essentially correct. You struggle a bit more than necessary showing the complement of the union you found is infinite.
    – Ethan Bolker
    Nov 18 at 13:52















up vote
1
down vote

favorite












Let $X$ be a not empty set.



I must prove that the family $mathcal{A}$ defined as follows
$$mathcal{A}=bigg{Ainmathcal{P}(X);|;A;text{is finite or $A^c$ is finite}bigg}$$ it is not a $sigma$-algebra if $X$ is infinite.



Obviously $mathcal{A}$ is an algebra. To show that it is not a $sigma$-algebra I reasoned as follows: fixed a sequence ${x_n}_{ninmathbb{N}}$ of elements of $X$ such that $x_ine x_j$ for $ine j$ and let $A_n$ be the set consisting of the single point $x_n$, in symbols $A_n={x_n}$ for all $ninmathbb{N}$. At this point I consider the sets $A_2,A_4dots ,A_{2k},dots$ where $kinmathbb{N}$; I observe that $A_{2k}inmathcal{A}$ for all $kinmathbb{N}$, but the union $$bigcup_{k=1}^{+infty}A_{2k}=bigg{x_2,x_4,dots,x_{2k},dotsbigg}$$
is infinite. On the other hand
begin{equation}
begin{split}
bigg[bigcup_{k=1}^{+infty}A_{2k}bigg]^c=Xsetminusbigcup_{k=1}^{+infty}A_{2k}=& Xcapbigg[bigcup_{k=1}^{+infty}A_{2k}bigg]^{c}=Xcapbigg[bigcup_{k=1}^{+infty}A_{2k+1}bigg]\
=&bigcup_{k=1}^{+infty}A_{2k+1}cuptilde{A},
end{split}
end{equation}

where $tilde{A}$ it is what remains if $X$ is not countable, if $X$ is countable $tilde{A}=emptyset$. Therefore $cup_k A_{2k}$ and $[cup_k A_{2k}]^c$ are both infinite, then $$bigcup_{n=1}^{+infty} A_nnotinmathcal{A}.$$



it's correct? Thanks!










share|cite|improve this question






















  • Your solution is perfectly fine, you are using that a countable collection of sets in a sigma algebra stays within the sigma algebra, which is part of the definition of a sigma algebra and you are right in proving that this doesn’t hold here
    – Sorin Tirc
    Nov 18 at 13:52










  • Your argument is essentially correct. You struggle a bit more than necessary showing the complement of the union you found is infinite.
    – Ethan Bolker
    Nov 18 at 13:52













up vote
1
down vote

favorite









up vote
1
down vote

favorite











Let $X$ be a not empty set.



I must prove that the family $mathcal{A}$ defined as follows
$$mathcal{A}=bigg{Ainmathcal{P}(X);|;A;text{is finite or $A^c$ is finite}bigg}$$ it is not a $sigma$-algebra if $X$ is infinite.



Obviously $mathcal{A}$ is an algebra. To show that it is not a $sigma$-algebra I reasoned as follows: fixed a sequence ${x_n}_{ninmathbb{N}}$ of elements of $X$ such that $x_ine x_j$ for $ine j$ and let $A_n$ be the set consisting of the single point $x_n$, in symbols $A_n={x_n}$ for all $ninmathbb{N}$. At this point I consider the sets $A_2,A_4dots ,A_{2k},dots$ where $kinmathbb{N}$; I observe that $A_{2k}inmathcal{A}$ for all $kinmathbb{N}$, but the union $$bigcup_{k=1}^{+infty}A_{2k}=bigg{x_2,x_4,dots,x_{2k},dotsbigg}$$
is infinite. On the other hand
begin{equation}
begin{split}
bigg[bigcup_{k=1}^{+infty}A_{2k}bigg]^c=Xsetminusbigcup_{k=1}^{+infty}A_{2k}=& Xcapbigg[bigcup_{k=1}^{+infty}A_{2k}bigg]^{c}=Xcapbigg[bigcup_{k=1}^{+infty}A_{2k+1}bigg]\
=&bigcup_{k=1}^{+infty}A_{2k+1}cuptilde{A},
end{split}
end{equation}

where $tilde{A}$ it is what remains if $X$ is not countable, if $X$ is countable $tilde{A}=emptyset$. Therefore $cup_k A_{2k}$ and $[cup_k A_{2k}]^c$ are both infinite, then $$bigcup_{n=1}^{+infty} A_nnotinmathcal{A}.$$



it's correct? Thanks!










share|cite|improve this question













Let $X$ be a not empty set.



I must prove that the family $mathcal{A}$ defined as follows
$$mathcal{A}=bigg{Ainmathcal{P}(X);|;A;text{is finite or $A^c$ is finite}bigg}$$ it is not a $sigma$-algebra if $X$ is infinite.



Obviously $mathcal{A}$ is an algebra. To show that it is not a $sigma$-algebra I reasoned as follows: fixed a sequence ${x_n}_{ninmathbb{N}}$ of elements of $X$ such that $x_ine x_j$ for $ine j$ and let $A_n$ be the set consisting of the single point $x_n$, in symbols $A_n={x_n}$ for all $ninmathbb{N}$. At this point I consider the sets $A_2,A_4dots ,A_{2k},dots$ where $kinmathbb{N}$; I observe that $A_{2k}inmathcal{A}$ for all $kinmathbb{N}$, but the union $$bigcup_{k=1}^{+infty}A_{2k}=bigg{x_2,x_4,dots,x_{2k},dotsbigg}$$
is infinite. On the other hand
begin{equation}
begin{split}
bigg[bigcup_{k=1}^{+infty}A_{2k}bigg]^c=Xsetminusbigcup_{k=1}^{+infty}A_{2k}=& Xcapbigg[bigcup_{k=1}^{+infty}A_{2k}bigg]^{c}=Xcapbigg[bigcup_{k=1}^{+infty}A_{2k+1}bigg]\
=&bigcup_{k=1}^{+infty}A_{2k+1}cuptilde{A},
end{split}
end{equation}

where $tilde{A}$ it is what remains if $X$ is not countable, if $X$ is countable $tilde{A}=emptyset$. Therefore $cup_k A_{2k}$ and $[cup_k A_{2k}]^c$ are both infinite, then $$bigcup_{n=1}^{+infty} A_nnotinmathcal{A}.$$



it's correct? Thanks!







proof-verification proof-writing proof-explanation






share|cite|improve this question













share|cite|improve this question











share|cite|improve this question




share|cite|improve this question










asked Nov 18 at 13:48









Jack J.

4441317




4441317












  • Your solution is perfectly fine, you are using that a countable collection of sets in a sigma algebra stays within the sigma algebra, which is part of the definition of a sigma algebra and you are right in proving that this doesn’t hold here
    – Sorin Tirc
    Nov 18 at 13:52










  • Your argument is essentially correct. You struggle a bit more than necessary showing the complement of the union you found is infinite.
    – Ethan Bolker
    Nov 18 at 13:52


















  • Your solution is perfectly fine, you are using that a countable collection of sets in a sigma algebra stays within the sigma algebra, which is part of the definition of a sigma algebra and you are right in proving that this doesn’t hold here
    – Sorin Tirc
    Nov 18 at 13:52










  • Your argument is essentially correct. You struggle a bit more than necessary showing the complement of the union you found is infinite.
    – Ethan Bolker
    Nov 18 at 13:52
















Your solution is perfectly fine, you are using that a countable collection of sets in a sigma algebra stays within the sigma algebra, which is part of the definition of a sigma algebra and you are right in proving that this doesn’t hold here
– Sorin Tirc
Nov 18 at 13:52




Your solution is perfectly fine, you are using that a countable collection of sets in a sigma algebra stays within the sigma algebra, which is part of the definition of a sigma algebra and you are right in proving that this doesn’t hold here
– Sorin Tirc
Nov 18 at 13:52












Your argument is essentially correct. You struggle a bit more than necessary showing the complement of the union you found is infinite.
– Ethan Bolker
Nov 18 at 13:52




Your argument is essentially correct. You struggle a bit more than necessary showing the complement of the union you found is infinite.
– Ethan Bolker
Nov 18 at 13:52










1 Answer
1






active

oldest

votes

















up vote
2
down vote



accepted










Your proof is correct.



A bit shorter:



Let $X$ be infinite.



Then we can write it as $X=Ycup Z$ where:





  • $Y$ and $Z$ are disjoint.


  • $Y$ is countably infinite.


  • $Z$ is infinite.


Then $Ynotinmathcal A$, but also $Y=bigcup_{yin Y}{y}$ where the RHS is a countable union of sets that are elements of $mathcal A$.



So $mathcal A$ is not closed under countable unions, hence is not a $sigma$-algebra.






share|cite|improve this answer























    Your Answer





    StackExchange.ifUsing("editor", function () {
    return StackExchange.using("mathjaxEditing", function () {
    StackExchange.MarkdownEditor.creationCallbacks.add(function (editor, postfix) {
    StackExchange.mathjaxEditing.prepareWmdForMathJax(editor, postfix, [["$", "$"], ["\\(","\\)"]]);
    });
    });
    }, "mathjax-editing");

    StackExchange.ready(function() {
    var channelOptions = {
    tags: "".split(" "),
    id: "69"
    };
    initTagRenderer("".split(" "), "".split(" "), channelOptions);

    StackExchange.using("externalEditor", function() {
    // Have to fire editor after snippets, if snippets enabled
    if (StackExchange.settings.snippets.snippetsEnabled) {
    StackExchange.using("snippets", function() {
    createEditor();
    });
    }
    else {
    createEditor();
    }
    });

    function createEditor() {
    StackExchange.prepareEditor({
    heartbeatType: 'answer',
    convertImagesToLinks: true,
    noModals: true,
    showLowRepImageUploadWarning: true,
    reputationToPostImages: 10,
    bindNavPrevention: true,
    postfix: "",
    imageUploader: {
    brandingHtml: "Powered by u003ca class="icon-imgur-white" href="https://imgur.com/"u003eu003c/au003e",
    contentPolicyHtml: "User contributions licensed under u003ca href="https://creativecommons.org/licenses/by-sa/3.0/"u003ecc by-sa 3.0 with attribution requiredu003c/au003e u003ca href="https://stackoverflow.com/legal/content-policy"u003e(content policy)u003c/au003e",
    allowUrls: true
    },
    noCode: true, onDemand: true,
    discardSelector: ".discard-answer"
    ,immediatelyShowMarkdownHelp:true
    });


    }
    });














    draft saved

    draft discarded


















    StackExchange.ready(
    function () {
    StackExchange.openid.initPostLogin('.new-post-login', 'https%3a%2f%2fmath.stackexchange.com%2fquestions%2f3003552%2fa-family-of-sets-that-is-not-a-sigma-algebra%23new-answer', 'question_page');
    }
    );

    Post as a guest















    Required, but never shown

























    1 Answer
    1






    active

    oldest

    votes








    1 Answer
    1






    active

    oldest

    votes









    active

    oldest

    votes






    active

    oldest

    votes








    up vote
    2
    down vote



    accepted










    Your proof is correct.



    A bit shorter:



    Let $X$ be infinite.



    Then we can write it as $X=Ycup Z$ where:





    • $Y$ and $Z$ are disjoint.


    • $Y$ is countably infinite.


    • $Z$ is infinite.


    Then $Ynotinmathcal A$, but also $Y=bigcup_{yin Y}{y}$ where the RHS is a countable union of sets that are elements of $mathcal A$.



    So $mathcal A$ is not closed under countable unions, hence is not a $sigma$-algebra.






    share|cite|improve this answer



























      up vote
      2
      down vote



      accepted










      Your proof is correct.



      A bit shorter:



      Let $X$ be infinite.



      Then we can write it as $X=Ycup Z$ where:





      • $Y$ and $Z$ are disjoint.


      • $Y$ is countably infinite.


      • $Z$ is infinite.


      Then $Ynotinmathcal A$, but also $Y=bigcup_{yin Y}{y}$ where the RHS is a countable union of sets that are elements of $mathcal A$.



      So $mathcal A$ is not closed under countable unions, hence is not a $sigma$-algebra.






      share|cite|improve this answer

























        up vote
        2
        down vote



        accepted







        up vote
        2
        down vote



        accepted






        Your proof is correct.



        A bit shorter:



        Let $X$ be infinite.



        Then we can write it as $X=Ycup Z$ where:





        • $Y$ and $Z$ are disjoint.


        • $Y$ is countably infinite.


        • $Z$ is infinite.


        Then $Ynotinmathcal A$, but also $Y=bigcup_{yin Y}{y}$ where the RHS is a countable union of sets that are elements of $mathcal A$.



        So $mathcal A$ is not closed under countable unions, hence is not a $sigma$-algebra.






        share|cite|improve this answer














        Your proof is correct.



        A bit shorter:



        Let $X$ be infinite.



        Then we can write it as $X=Ycup Z$ where:





        • $Y$ and $Z$ are disjoint.


        • $Y$ is countably infinite.


        • $Z$ is infinite.


        Then $Ynotinmathcal A$, but also $Y=bigcup_{yin Y}{y}$ where the RHS is a countable union of sets that are elements of $mathcal A$.



        So $mathcal A$ is not closed under countable unions, hence is not a $sigma$-algebra.







        share|cite|improve this answer














        share|cite|improve this answer



        share|cite|improve this answer








        edited Nov 18 at 20:37

























        answered Nov 18 at 13:59









        drhab

        94.9k543125




        94.9k543125






























            draft saved

            draft discarded




















































            Thanks for contributing an answer to Mathematics Stack Exchange!


            • Please be sure to answer the question. Provide details and share your research!

            But avoid



            • Asking for help, clarification, or responding to other answers.

            • Making statements based on opinion; back them up with references or personal experience.


            Use MathJax to format equations. MathJax reference.


            To learn more, see our tips on writing great answers.





            Some of your past answers have not been well-received, and you're in danger of being blocked from answering.


            Please pay close attention to the following guidance:


            • Please be sure to answer the question. Provide details and share your research!

            But avoid



            • Asking for help, clarification, or responding to other answers.

            • Making statements based on opinion; back them up with references or personal experience.


            To learn more, see our tips on writing great answers.




            draft saved


            draft discarded














            StackExchange.ready(
            function () {
            StackExchange.openid.initPostLogin('.new-post-login', 'https%3a%2f%2fmath.stackexchange.com%2fquestions%2f3003552%2fa-family-of-sets-that-is-not-a-sigma-algebra%23new-answer', 'question_page');
            }
            );

            Post as a guest















            Required, but never shown





















































            Required, but never shown














            Required, but never shown












            Required, but never shown







            Required, but never shown

































            Required, but never shown














            Required, but never shown












            Required, but never shown







            Required, but never shown







            Popular posts from this blog

            Quarter-circle Tiles

            build a pushdown automaton that recognizes the reverse language of a given pushdown automaton?

            Mont Emei